Prove that the product of any two numbers between two consecutive squares is never a perfect square












4












$begingroup$


In essence, I want to prove that the product of any two distinct elements in the set ${n^2, n^2+1, ... , (n+1)^2-1}$ is never a perfect square for a positive integers $n$. I have no idea on how to prove it, but I've also yet to find a counterexample to this statement. Can anyone help?










share|cite|improve this question







New contributor




Ed T is a new contributor to this site. Take care in asking for clarification, commenting, and answering.
Check out our Code of Conduct.







$endgroup$












  • $begingroup$
    just inequalities. All positive integers, can we have $n^2 < 2 w^2 < 2 (w+1)^2 < (n+1)^2 ; ; ? ; ;$ How about $n^2 < 3 w^2 < 3 (w+1)^2 < (n+1)^2 ; ; ? ; ;$
    $endgroup$
    – Will Jagy
    9 hours ago






  • 1




    $begingroup$
    is $n^2$ included in the set? If so, why is $(n+1)^2$ not included?
    $endgroup$
    – Dr. Mathva
    9 hours ago








  • 2




    $begingroup$
    @Dr.Mathva Because then the statement is obviously false, with (n(n+1))^2
    $endgroup$
    – gnasher729
    9 hours ago
















4












$begingroup$


In essence, I want to prove that the product of any two distinct elements in the set ${n^2, n^2+1, ... , (n+1)^2-1}$ is never a perfect square for a positive integers $n$. I have no idea on how to prove it, but I've also yet to find a counterexample to this statement. Can anyone help?










share|cite|improve this question







New contributor




Ed T is a new contributor to this site. Take care in asking for clarification, commenting, and answering.
Check out our Code of Conduct.







$endgroup$












  • $begingroup$
    just inequalities. All positive integers, can we have $n^2 < 2 w^2 < 2 (w+1)^2 < (n+1)^2 ; ; ? ; ;$ How about $n^2 < 3 w^2 < 3 (w+1)^2 < (n+1)^2 ; ; ? ; ;$
    $endgroup$
    – Will Jagy
    9 hours ago






  • 1




    $begingroup$
    is $n^2$ included in the set? If so, why is $(n+1)^2$ not included?
    $endgroup$
    – Dr. Mathva
    9 hours ago








  • 2




    $begingroup$
    @Dr.Mathva Because then the statement is obviously false, with (n(n+1))^2
    $endgroup$
    – gnasher729
    9 hours ago














4












4








4


2



$begingroup$


In essence, I want to prove that the product of any two distinct elements in the set ${n^2, n^2+1, ... , (n+1)^2-1}$ is never a perfect square for a positive integers $n$. I have no idea on how to prove it, but I've also yet to find a counterexample to this statement. Can anyone help?










share|cite|improve this question







New contributor




Ed T is a new contributor to this site. Take care in asking for clarification, commenting, and answering.
Check out our Code of Conduct.







$endgroup$




In essence, I want to prove that the product of any two distinct elements in the set ${n^2, n^2+1, ... , (n+1)^2-1}$ is never a perfect square for a positive integers $n$. I have no idea on how to prove it, but I've also yet to find a counterexample to this statement. Can anyone help?







number-theory elementary-number-theory






share|cite|improve this question







New contributor




Ed T is a new contributor to this site. Take care in asking for clarification, commenting, and answering.
Check out our Code of Conduct.











share|cite|improve this question







New contributor




Ed T is a new contributor to this site. Take care in asking for clarification, commenting, and answering.
Check out our Code of Conduct.









share|cite|improve this question




share|cite|improve this question






New contributor




Ed T is a new contributor to this site. Take care in asking for clarification, commenting, and answering.
Check out our Code of Conduct.









asked 10 hours ago









Ed TEd T

211




211




New contributor




Ed T is a new contributor to this site. Take care in asking for clarification, commenting, and answering.
Check out our Code of Conduct.





New contributor





Ed T is a new contributor to this site. Take care in asking for clarification, commenting, and answering.
Check out our Code of Conduct.






Ed T is a new contributor to this site. Take care in asking for clarification, commenting, and answering.
Check out our Code of Conduct.












  • $begingroup$
    just inequalities. All positive integers, can we have $n^2 < 2 w^2 < 2 (w+1)^2 < (n+1)^2 ; ; ? ; ;$ How about $n^2 < 3 w^2 < 3 (w+1)^2 < (n+1)^2 ; ; ? ; ;$
    $endgroup$
    – Will Jagy
    9 hours ago






  • 1




    $begingroup$
    is $n^2$ included in the set? If so, why is $(n+1)^2$ not included?
    $endgroup$
    – Dr. Mathva
    9 hours ago








  • 2




    $begingroup$
    @Dr.Mathva Because then the statement is obviously false, with (n(n+1))^2
    $endgroup$
    – gnasher729
    9 hours ago


















  • $begingroup$
    just inequalities. All positive integers, can we have $n^2 < 2 w^2 < 2 (w+1)^2 < (n+1)^2 ; ; ? ; ;$ How about $n^2 < 3 w^2 < 3 (w+1)^2 < (n+1)^2 ; ; ? ; ;$
    $endgroup$
    – Will Jagy
    9 hours ago






  • 1




    $begingroup$
    is $n^2$ included in the set? If so, why is $(n+1)^2$ not included?
    $endgroup$
    – Dr. Mathva
    9 hours ago








  • 2




    $begingroup$
    @Dr.Mathva Because then the statement is obviously false, with (n(n+1))^2
    $endgroup$
    – gnasher729
    9 hours ago
















$begingroup$
just inequalities. All positive integers, can we have $n^2 < 2 w^2 < 2 (w+1)^2 < (n+1)^2 ; ; ? ; ;$ How about $n^2 < 3 w^2 < 3 (w+1)^2 < (n+1)^2 ; ; ? ; ;$
$endgroup$
– Will Jagy
9 hours ago




$begingroup$
just inequalities. All positive integers, can we have $n^2 < 2 w^2 < 2 (w+1)^2 < (n+1)^2 ; ; ? ; ;$ How about $n^2 < 3 w^2 < 3 (w+1)^2 < (n+1)^2 ; ; ? ; ;$
$endgroup$
– Will Jagy
9 hours ago




1




1




$begingroup$
is $n^2$ included in the set? If so, why is $(n+1)^2$ not included?
$endgroup$
– Dr. Mathva
9 hours ago






$begingroup$
is $n^2$ included in the set? If so, why is $(n+1)^2$ not included?
$endgroup$
– Dr. Mathva
9 hours ago






2




2




$begingroup$
@Dr.Mathva Because then the statement is obviously false, with (n(n+1))^2
$endgroup$
– gnasher729
9 hours ago




$begingroup$
@Dr.Mathva Because then the statement is obviously false, with (n(n+1))^2
$endgroup$
– gnasher729
9 hours ago










2 Answers
2






active

oldest

votes


















5












$begingroup$

First, note that $n^2$ can't be one of the elements as the other element would also need to be a perfect square for the product to be a perfect square. As gnasher729 commented to the answer, this is why $left(n + 1right)^2$ is not included.



Assume there are $2$ such elements, $n^2 + a$ and $n^2 + b$, with $a neq b$, $1 le a, b le 2n$ and, WLOG, $a lt b$. Thus, consider their product to be a perfect square of some integer $p$, i.e.,



$$left(n^2 + aright)left(n^2 + bright) = p^2 tag{1}label{eq1}$$



As $n^2 + a lt p lt n^2 + b$, let



$$p = n^2 + c tag{2}label{eq2}$$



for some $a lt c lt b$. Thus, for some positive integers $d$ and $e$, we have that



$$a = c - d tag{3}label{eq3}$$
$$b = c + e tag{4}label{eq4}$$



Substitute eqref{eq2}, eqref{eq3} and eqref{eq4} into eqref{eq1} to get



$$left(n^2 + left(c - dright)right)left(n^2 + left(c + eright)right) = left(n^2 + cright)^2 tag{5}label{eq5}$$



Expanding both sides gives



$$n^4 + 2cn^2 + left(e - dright)n^2 + c^2 + cleft(e - dright) - ed = n^4 + 2cn^2 + c^2 tag{6}label{eq6}$$



Removing the common terms on both sides and moving the remaining terms, apart from the $n^2$ one, to the right gives



$$left(e - dright)n^2 = -cleft(e - dright) + ed tag{7}label{eq7}$$



Note that $e le d$ won't work because the LHS would be become non-positive but the RHS would become positive. Thus, consider $e gt d$, i.e., let



$$e = d + m, text{ with } m ge 1 tag{8}label{eq8}$$



Thus,



$$e + d lt 2n Rightarrow 2d + m lt 2n Rightarrow d lt n - frac{m}{2} tag{9}label{eq9}$$



Also,



$$ed = left(d + mright)d lt left(n + frac{m}{2}right)left(n - frac{m}{2}right) = n^2 - frac{m^2}{4} lt n^2 tag{10}label{eq10}$$



As such, the RHS of eqref{eq7} is $lt n^2$, so it can't be true.






share|cite|improve this answer









$endgroup$





















    2












    $begingroup$

    For any two numbers $n^2+a, n^2+b; 0<a<b<(2n+1)$, their product will satisfy $n^4<n^4+(a+b)n^2+ab<(n^2+2n+1)^2$.



    All of the squares between $n^4$ and $(n^2+2n+1)^2$ will have the form $(n^2+m)^2=n^4+2mn^2+m^2; 1le mle 2n$



    If $n^4+(a+b)n^2+ab$ is a perfect square, it will be one of the squares between $n^4$ and $(n^2+2n+1)^2$ and hence equal to $n^4+2mn^2+m^2$ for some $m$.



    Thus $(a+b)=2m; ab=m^2$ for some $m$



    Rearranging, we get $m^2=frac{a^2+2ab+b^2}{4}=ab=frac{4ab}{4}$, or $a^2+b^2=2ab$.



    This implies both $amid b$ and $bmid a$, meaning $b=a$ and the numbers being multiplied to obtain a perfect square are not distinct.






    share|cite|improve this answer









    $endgroup$









    • 1




      $begingroup$
      Simpler: if $a+2+b^2 = 2ab$ then $(a-b)^2 = 0$ so $a = b$. Nice answer.
      $endgroup$
      – marty cohen
      8 hours ago






    • 3




      $begingroup$
      Note that $n$ is a fixed value. How do you know that, for example, $a + b = 2m - 1$ and $ab = n^2 + m^2$ can't be true instead? It might be intuitively obvious to you, & others, but it's not to me. This is why I try to explicitly show this doesn't occur in my answer.
      $endgroup$
      – John Omielan
      8 hours ago













    Your Answer





    StackExchange.ifUsing("editor", function () {
    return StackExchange.using("mathjaxEditing", function () {
    StackExchange.MarkdownEditor.creationCallbacks.add(function (editor, postfix) {
    StackExchange.mathjaxEditing.prepareWmdForMathJax(editor, postfix, [["$", "$"], ["\\(","\\)"]]);
    });
    });
    }, "mathjax-editing");

    StackExchange.ready(function() {
    var channelOptions = {
    tags: "".split(" "),
    id: "69"
    };
    initTagRenderer("".split(" "), "".split(" "), channelOptions);

    StackExchange.using("externalEditor", function() {
    // Have to fire editor after snippets, if snippets enabled
    if (StackExchange.settings.snippets.snippetsEnabled) {
    StackExchange.using("snippets", function() {
    createEditor();
    });
    }
    else {
    createEditor();
    }
    });

    function createEditor() {
    StackExchange.prepareEditor({
    heartbeatType: 'answer',
    autoActivateHeartbeat: false,
    convertImagesToLinks: true,
    noModals: true,
    showLowRepImageUploadWarning: true,
    reputationToPostImages: 10,
    bindNavPrevention: true,
    postfix: "",
    imageUploader: {
    brandingHtml: "Powered by u003ca class="icon-imgur-white" href="https://imgur.com/"u003eu003c/au003e",
    contentPolicyHtml: "User contributions licensed under u003ca href="https://creativecommons.org/licenses/by-sa/3.0/"u003ecc by-sa 3.0 with attribution requiredu003c/au003e u003ca href="https://stackoverflow.com/legal/content-policy"u003e(content policy)u003c/au003e",
    allowUrls: true
    },
    noCode: true, onDemand: true,
    discardSelector: ".discard-answer"
    ,immediatelyShowMarkdownHelp:true
    });


    }
    });






    Ed T is a new contributor. Be nice, and check out our Code of Conduct.










    draft saved

    draft discarded


















    StackExchange.ready(
    function () {
    StackExchange.openid.initPostLogin('.new-post-login', 'https%3a%2f%2fmath.stackexchange.com%2fquestions%2f3116686%2fprove-that-the-product-of-any-two-numbers-between-two-consecutive-squares-is-nev%23new-answer', 'question_page');
    }
    );

    Post as a guest















    Required, but never shown

























    2 Answers
    2






    active

    oldest

    votes








    2 Answers
    2






    active

    oldest

    votes









    active

    oldest

    votes






    active

    oldest

    votes









    5












    $begingroup$

    First, note that $n^2$ can't be one of the elements as the other element would also need to be a perfect square for the product to be a perfect square. As gnasher729 commented to the answer, this is why $left(n + 1right)^2$ is not included.



    Assume there are $2$ such elements, $n^2 + a$ and $n^2 + b$, with $a neq b$, $1 le a, b le 2n$ and, WLOG, $a lt b$. Thus, consider their product to be a perfect square of some integer $p$, i.e.,



    $$left(n^2 + aright)left(n^2 + bright) = p^2 tag{1}label{eq1}$$



    As $n^2 + a lt p lt n^2 + b$, let



    $$p = n^2 + c tag{2}label{eq2}$$



    for some $a lt c lt b$. Thus, for some positive integers $d$ and $e$, we have that



    $$a = c - d tag{3}label{eq3}$$
    $$b = c + e tag{4}label{eq4}$$



    Substitute eqref{eq2}, eqref{eq3} and eqref{eq4} into eqref{eq1} to get



    $$left(n^2 + left(c - dright)right)left(n^2 + left(c + eright)right) = left(n^2 + cright)^2 tag{5}label{eq5}$$



    Expanding both sides gives



    $$n^4 + 2cn^2 + left(e - dright)n^2 + c^2 + cleft(e - dright) - ed = n^4 + 2cn^2 + c^2 tag{6}label{eq6}$$



    Removing the common terms on both sides and moving the remaining terms, apart from the $n^2$ one, to the right gives



    $$left(e - dright)n^2 = -cleft(e - dright) + ed tag{7}label{eq7}$$



    Note that $e le d$ won't work because the LHS would be become non-positive but the RHS would become positive. Thus, consider $e gt d$, i.e., let



    $$e = d + m, text{ with } m ge 1 tag{8}label{eq8}$$



    Thus,



    $$e + d lt 2n Rightarrow 2d + m lt 2n Rightarrow d lt n - frac{m}{2} tag{9}label{eq9}$$



    Also,



    $$ed = left(d + mright)d lt left(n + frac{m}{2}right)left(n - frac{m}{2}right) = n^2 - frac{m^2}{4} lt n^2 tag{10}label{eq10}$$



    As such, the RHS of eqref{eq7} is $lt n^2$, so it can't be true.






    share|cite|improve this answer









    $endgroup$


















      5












      $begingroup$

      First, note that $n^2$ can't be one of the elements as the other element would also need to be a perfect square for the product to be a perfect square. As gnasher729 commented to the answer, this is why $left(n + 1right)^2$ is not included.



      Assume there are $2$ such elements, $n^2 + a$ and $n^2 + b$, with $a neq b$, $1 le a, b le 2n$ and, WLOG, $a lt b$. Thus, consider their product to be a perfect square of some integer $p$, i.e.,



      $$left(n^2 + aright)left(n^2 + bright) = p^2 tag{1}label{eq1}$$



      As $n^2 + a lt p lt n^2 + b$, let



      $$p = n^2 + c tag{2}label{eq2}$$



      for some $a lt c lt b$. Thus, for some positive integers $d$ and $e$, we have that



      $$a = c - d tag{3}label{eq3}$$
      $$b = c + e tag{4}label{eq4}$$



      Substitute eqref{eq2}, eqref{eq3} and eqref{eq4} into eqref{eq1} to get



      $$left(n^2 + left(c - dright)right)left(n^2 + left(c + eright)right) = left(n^2 + cright)^2 tag{5}label{eq5}$$



      Expanding both sides gives



      $$n^4 + 2cn^2 + left(e - dright)n^2 + c^2 + cleft(e - dright) - ed = n^4 + 2cn^2 + c^2 tag{6}label{eq6}$$



      Removing the common terms on both sides and moving the remaining terms, apart from the $n^2$ one, to the right gives



      $$left(e - dright)n^2 = -cleft(e - dright) + ed tag{7}label{eq7}$$



      Note that $e le d$ won't work because the LHS would be become non-positive but the RHS would become positive. Thus, consider $e gt d$, i.e., let



      $$e = d + m, text{ with } m ge 1 tag{8}label{eq8}$$



      Thus,



      $$e + d lt 2n Rightarrow 2d + m lt 2n Rightarrow d lt n - frac{m}{2} tag{9}label{eq9}$$



      Also,



      $$ed = left(d + mright)d lt left(n + frac{m}{2}right)left(n - frac{m}{2}right) = n^2 - frac{m^2}{4} lt n^2 tag{10}label{eq10}$$



      As such, the RHS of eqref{eq7} is $lt n^2$, so it can't be true.






      share|cite|improve this answer









      $endgroup$
















        5












        5








        5





        $begingroup$

        First, note that $n^2$ can't be one of the elements as the other element would also need to be a perfect square for the product to be a perfect square. As gnasher729 commented to the answer, this is why $left(n + 1right)^2$ is not included.



        Assume there are $2$ such elements, $n^2 + a$ and $n^2 + b$, with $a neq b$, $1 le a, b le 2n$ and, WLOG, $a lt b$. Thus, consider their product to be a perfect square of some integer $p$, i.e.,



        $$left(n^2 + aright)left(n^2 + bright) = p^2 tag{1}label{eq1}$$



        As $n^2 + a lt p lt n^2 + b$, let



        $$p = n^2 + c tag{2}label{eq2}$$



        for some $a lt c lt b$. Thus, for some positive integers $d$ and $e$, we have that



        $$a = c - d tag{3}label{eq3}$$
        $$b = c + e tag{4}label{eq4}$$



        Substitute eqref{eq2}, eqref{eq3} and eqref{eq4} into eqref{eq1} to get



        $$left(n^2 + left(c - dright)right)left(n^2 + left(c + eright)right) = left(n^2 + cright)^2 tag{5}label{eq5}$$



        Expanding both sides gives



        $$n^4 + 2cn^2 + left(e - dright)n^2 + c^2 + cleft(e - dright) - ed = n^4 + 2cn^2 + c^2 tag{6}label{eq6}$$



        Removing the common terms on both sides and moving the remaining terms, apart from the $n^2$ one, to the right gives



        $$left(e - dright)n^2 = -cleft(e - dright) + ed tag{7}label{eq7}$$



        Note that $e le d$ won't work because the LHS would be become non-positive but the RHS would become positive. Thus, consider $e gt d$, i.e., let



        $$e = d + m, text{ with } m ge 1 tag{8}label{eq8}$$



        Thus,



        $$e + d lt 2n Rightarrow 2d + m lt 2n Rightarrow d lt n - frac{m}{2} tag{9}label{eq9}$$



        Also,



        $$ed = left(d + mright)d lt left(n + frac{m}{2}right)left(n - frac{m}{2}right) = n^2 - frac{m^2}{4} lt n^2 tag{10}label{eq10}$$



        As such, the RHS of eqref{eq7} is $lt n^2$, so it can't be true.






        share|cite|improve this answer









        $endgroup$



        First, note that $n^2$ can't be one of the elements as the other element would also need to be a perfect square for the product to be a perfect square. As gnasher729 commented to the answer, this is why $left(n + 1right)^2$ is not included.



        Assume there are $2$ such elements, $n^2 + a$ and $n^2 + b$, with $a neq b$, $1 le a, b le 2n$ and, WLOG, $a lt b$. Thus, consider their product to be a perfect square of some integer $p$, i.e.,



        $$left(n^2 + aright)left(n^2 + bright) = p^2 tag{1}label{eq1}$$



        As $n^2 + a lt p lt n^2 + b$, let



        $$p = n^2 + c tag{2}label{eq2}$$



        for some $a lt c lt b$. Thus, for some positive integers $d$ and $e$, we have that



        $$a = c - d tag{3}label{eq3}$$
        $$b = c + e tag{4}label{eq4}$$



        Substitute eqref{eq2}, eqref{eq3} and eqref{eq4} into eqref{eq1} to get



        $$left(n^2 + left(c - dright)right)left(n^2 + left(c + eright)right) = left(n^2 + cright)^2 tag{5}label{eq5}$$



        Expanding both sides gives



        $$n^4 + 2cn^2 + left(e - dright)n^2 + c^2 + cleft(e - dright) - ed = n^4 + 2cn^2 + c^2 tag{6}label{eq6}$$



        Removing the common terms on both sides and moving the remaining terms, apart from the $n^2$ one, to the right gives



        $$left(e - dright)n^2 = -cleft(e - dright) + ed tag{7}label{eq7}$$



        Note that $e le d$ won't work because the LHS would be become non-positive but the RHS would become positive. Thus, consider $e gt d$, i.e., let



        $$e = d + m, text{ with } m ge 1 tag{8}label{eq8}$$



        Thus,



        $$e + d lt 2n Rightarrow 2d + m lt 2n Rightarrow d lt n - frac{m}{2} tag{9}label{eq9}$$



        Also,



        $$ed = left(d + mright)d lt left(n + frac{m}{2}right)left(n - frac{m}{2}right) = n^2 - frac{m^2}{4} lt n^2 tag{10}label{eq10}$$



        As such, the RHS of eqref{eq7} is $lt n^2$, so it can't be true.







        share|cite|improve this answer












        share|cite|improve this answer



        share|cite|improve this answer










        answered 8 hours ago









        John OmielanJohn Omielan

        2,864212




        2,864212























            2












            $begingroup$

            For any two numbers $n^2+a, n^2+b; 0<a<b<(2n+1)$, their product will satisfy $n^4<n^4+(a+b)n^2+ab<(n^2+2n+1)^2$.



            All of the squares between $n^4$ and $(n^2+2n+1)^2$ will have the form $(n^2+m)^2=n^4+2mn^2+m^2; 1le mle 2n$



            If $n^4+(a+b)n^2+ab$ is a perfect square, it will be one of the squares between $n^4$ and $(n^2+2n+1)^2$ and hence equal to $n^4+2mn^2+m^2$ for some $m$.



            Thus $(a+b)=2m; ab=m^2$ for some $m$



            Rearranging, we get $m^2=frac{a^2+2ab+b^2}{4}=ab=frac{4ab}{4}$, or $a^2+b^2=2ab$.



            This implies both $amid b$ and $bmid a$, meaning $b=a$ and the numbers being multiplied to obtain a perfect square are not distinct.






            share|cite|improve this answer









            $endgroup$









            • 1




              $begingroup$
              Simpler: if $a+2+b^2 = 2ab$ then $(a-b)^2 = 0$ so $a = b$. Nice answer.
              $endgroup$
              – marty cohen
              8 hours ago






            • 3




              $begingroup$
              Note that $n$ is a fixed value. How do you know that, for example, $a + b = 2m - 1$ and $ab = n^2 + m^2$ can't be true instead? It might be intuitively obvious to you, & others, but it's not to me. This is why I try to explicitly show this doesn't occur in my answer.
              $endgroup$
              – John Omielan
              8 hours ago


















            2












            $begingroup$

            For any two numbers $n^2+a, n^2+b; 0<a<b<(2n+1)$, their product will satisfy $n^4<n^4+(a+b)n^2+ab<(n^2+2n+1)^2$.



            All of the squares between $n^4$ and $(n^2+2n+1)^2$ will have the form $(n^2+m)^2=n^4+2mn^2+m^2; 1le mle 2n$



            If $n^4+(a+b)n^2+ab$ is a perfect square, it will be one of the squares between $n^4$ and $(n^2+2n+1)^2$ and hence equal to $n^4+2mn^2+m^2$ for some $m$.



            Thus $(a+b)=2m; ab=m^2$ for some $m$



            Rearranging, we get $m^2=frac{a^2+2ab+b^2}{4}=ab=frac{4ab}{4}$, or $a^2+b^2=2ab$.



            This implies both $amid b$ and $bmid a$, meaning $b=a$ and the numbers being multiplied to obtain a perfect square are not distinct.






            share|cite|improve this answer









            $endgroup$









            • 1




              $begingroup$
              Simpler: if $a+2+b^2 = 2ab$ then $(a-b)^2 = 0$ so $a = b$. Nice answer.
              $endgroup$
              – marty cohen
              8 hours ago






            • 3




              $begingroup$
              Note that $n$ is a fixed value. How do you know that, for example, $a + b = 2m - 1$ and $ab = n^2 + m^2$ can't be true instead? It might be intuitively obvious to you, & others, but it's not to me. This is why I try to explicitly show this doesn't occur in my answer.
              $endgroup$
              – John Omielan
              8 hours ago
















            2












            2








            2





            $begingroup$

            For any two numbers $n^2+a, n^2+b; 0<a<b<(2n+1)$, their product will satisfy $n^4<n^4+(a+b)n^2+ab<(n^2+2n+1)^2$.



            All of the squares between $n^4$ and $(n^2+2n+1)^2$ will have the form $(n^2+m)^2=n^4+2mn^2+m^2; 1le mle 2n$



            If $n^4+(a+b)n^2+ab$ is a perfect square, it will be one of the squares between $n^4$ and $(n^2+2n+1)^2$ and hence equal to $n^4+2mn^2+m^2$ for some $m$.



            Thus $(a+b)=2m; ab=m^2$ for some $m$



            Rearranging, we get $m^2=frac{a^2+2ab+b^2}{4}=ab=frac{4ab}{4}$, or $a^2+b^2=2ab$.



            This implies both $amid b$ and $bmid a$, meaning $b=a$ and the numbers being multiplied to obtain a perfect square are not distinct.






            share|cite|improve this answer









            $endgroup$



            For any two numbers $n^2+a, n^2+b; 0<a<b<(2n+1)$, their product will satisfy $n^4<n^4+(a+b)n^2+ab<(n^2+2n+1)^2$.



            All of the squares between $n^4$ and $(n^2+2n+1)^2$ will have the form $(n^2+m)^2=n^4+2mn^2+m^2; 1le mle 2n$



            If $n^4+(a+b)n^2+ab$ is a perfect square, it will be one of the squares between $n^4$ and $(n^2+2n+1)^2$ and hence equal to $n^4+2mn^2+m^2$ for some $m$.



            Thus $(a+b)=2m; ab=m^2$ for some $m$



            Rearranging, we get $m^2=frac{a^2+2ab+b^2}{4}=ab=frac{4ab}{4}$, or $a^2+b^2=2ab$.



            This implies both $amid b$ and $bmid a$, meaning $b=a$ and the numbers being multiplied to obtain a perfect square are not distinct.







            share|cite|improve this answer












            share|cite|improve this answer



            share|cite|improve this answer










            answered 8 hours ago









            Keith BackmanKeith Backman

            1,3681812




            1,3681812








            • 1




              $begingroup$
              Simpler: if $a+2+b^2 = 2ab$ then $(a-b)^2 = 0$ so $a = b$. Nice answer.
              $endgroup$
              – marty cohen
              8 hours ago






            • 3




              $begingroup$
              Note that $n$ is a fixed value. How do you know that, for example, $a + b = 2m - 1$ and $ab = n^2 + m^2$ can't be true instead? It might be intuitively obvious to you, & others, but it's not to me. This is why I try to explicitly show this doesn't occur in my answer.
              $endgroup$
              – John Omielan
              8 hours ago
















            • 1




              $begingroup$
              Simpler: if $a+2+b^2 = 2ab$ then $(a-b)^2 = 0$ so $a = b$. Nice answer.
              $endgroup$
              – marty cohen
              8 hours ago






            • 3




              $begingroup$
              Note that $n$ is a fixed value. How do you know that, for example, $a + b = 2m - 1$ and $ab = n^2 + m^2$ can't be true instead? It might be intuitively obvious to you, & others, but it's not to me. This is why I try to explicitly show this doesn't occur in my answer.
              $endgroup$
              – John Omielan
              8 hours ago










            1




            1




            $begingroup$
            Simpler: if $a+2+b^2 = 2ab$ then $(a-b)^2 = 0$ so $a = b$. Nice answer.
            $endgroup$
            – marty cohen
            8 hours ago




            $begingroup$
            Simpler: if $a+2+b^2 = 2ab$ then $(a-b)^2 = 0$ so $a = b$. Nice answer.
            $endgroup$
            – marty cohen
            8 hours ago




            3




            3




            $begingroup$
            Note that $n$ is a fixed value. How do you know that, for example, $a + b = 2m - 1$ and $ab = n^2 + m^2$ can't be true instead? It might be intuitively obvious to you, & others, but it's not to me. This is why I try to explicitly show this doesn't occur in my answer.
            $endgroup$
            – John Omielan
            8 hours ago






            $begingroup$
            Note that $n$ is a fixed value. How do you know that, for example, $a + b = 2m - 1$ and $ab = n^2 + m^2$ can't be true instead? It might be intuitively obvious to you, & others, but it's not to me. This is why I try to explicitly show this doesn't occur in my answer.
            $endgroup$
            – John Omielan
            8 hours ago












            Ed T is a new contributor. Be nice, and check out our Code of Conduct.










            draft saved

            draft discarded


















            Ed T is a new contributor. Be nice, and check out our Code of Conduct.













            Ed T is a new contributor. Be nice, and check out our Code of Conduct.












            Ed T is a new contributor. Be nice, and check out our Code of Conduct.
















            Thanks for contributing an answer to Mathematics Stack Exchange!


            • Please be sure to answer the question. Provide details and share your research!

            But avoid



            • Asking for help, clarification, or responding to other answers.

            • Making statements based on opinion; back them up with references or personal experience.


            Use MathJax to format equations. MathJax reference.


            To learn more, see our tips on writing great answers.




            draft saved


            draft discarded














            StackExchange.ready(
            function () {
            StackExchange.openid.initPostLogin('.new-post-login', 'https%3a%2f%2fmath.stackexchange.com%2fquestions%2f3116686%2fprove-that-the-product-of-any-two-numbers-between-two-consecutive-squares-is-nev%23new-answer', 'question_page');
            }
            );

            Post as a guest















            Required, but never shown





















































            Required, but never shown














            Required, but never shown












            Required, but never shown







            Required, but never shown

































            Required, but never shown














            Required, but never shown












            Required, but never shown







            Required, but never shown







            Popular posts from this blog

            Hivernacle

            Fluorita

            Hulsita